Sunteți pe pagina 1din 84

2016 Sports Medicine

Self-Assessment Examination
AAOS 2016 Sports medicine

Figure 1a Figure 1b

Figure 2a Figure 2b

1
AAOS 2016 Sports medicine

CLINICAL SITUATION FOR QUESTIONS 1 THROUGH 2


A 16-year-old boy fell while playing soccer. He said it felt like his knee buckled when he
planted his leg to kick a ball. He noticed an obvious deformity of his knee, which
spontaneously resolved with a clunk. He could not finish the game, but was able to bear
weight with a limp. He had 2 similar past episodes, but has never sought medical attention. An
initial examination demonstrated an effusion, tenderness at the proximal medial collateral
region and medial patellofemoral retinaculum, decreased range of motion, and patella
apprehension. A lateral patellar glide performed at 30 degrees of flexion was 3+. He was
otherwise ligamentously stable, and there were no other noteworthy findings.

Question 1 of 100
What do Figures 1a and 1b reveal?
1- Medial femoral condyle physeal widening
2- An ossseous or osteochondral loose fragment
3- Osgood-Schlatter disease
4- A patella nondisplaced fracture

PREFERRED RESPONSE: 2- An ossseous or osteochondral loose fragment

Question 2 of 100
Figures 2a and 2b are this patients proton density fat-saturated MR images. His tibial
tubercle-trochlear groove (TT-TG) distance is 12 mm, and he has normal limb-alignment film
findings. Treatment at this stage should include
1- hinged knee bracing, protected weight bearing, and physical therapy.
2- anteromedialization of the tibial tubercle.
3- internal fixation and medial patellofemoral ligament (MPFL) reconstruction.
4- arthroscopic lateral retinacular release.

PREFERRED RESPONSE: 3- internal fixation and medial patellofemoral ligament


(MPFL) reconstruction.

2
AAOS 2016 Sports medicine

DISCUSSION
This patients examination and history indicate recurrent patellar dislocations.
Radiographs show an osseous or osteochondral loose fragment. There is no evidence of an
obvious nondisplaced fracture or physeal changes. In the setting of suspected patella
dislocation or subluxation with loose fragment seen on radiograph, an MRI is indicated.
Lateral release alone is seldom indicated in a knee that is normal before injury. The
examination and MRI do not indicate a need for medial collateral ligament repair. Treatment
should consist of arthroscopy or arthrotomy and attempted internal fixation of this fragment.
If fixation is not possible, the loose body can be removed. Normal TT-TG values, an increased
lateral patellar glide, and a history of recurrent patellar dislocations after trauma suggest MPFL
incompetence and the need for reconstruction.

Question 3 of 100
Heat transfer from the skin to the environment when the ambient temperature exceeds
35C primarily is attributable to
1- evaporation.
2- conduction.
3- convection.
4- radiation.

PREFERRED RESPONSE: 1- evaporation.

DISCUSSION
Heat transfer from the skin to the environment occurs through conduction, convection,
evaporation, and radiation. Evaporation of sweat is the primary mechanism by which core
body temperature is regulated when the ambient temperature exceeds 35C. High humidity
can inhibit the evaporation of sweat, placing athletes at increased risk for heat-related illness,
which is defined as a core temperature above 40C. Symptoms include dizziness, confusion,
irritability, hyperventilation, nausea, vomiting, fatigue, and collapse. Initial treatment involves
rapid cooling through immersion in cold or ice water to prevent end-stage organ failure.

Question 4 of 100
Which factor increases the success rate associated with all-inside lateral meniscal repair?
1- Concomitant anterior cruciate ligament (ACL) reconstruction
2- Concomitant medial meniscus repair

3
AAOS 2016 Sports medicine

3- Older patient age


4- Varus knee alignment

PREFERRED RESPONSE: 1- Concomitant anterior cruciate ligament (ACL)


reconstruction

DISCUSSION
Decreased patient age, neutral alignment, and a concomitant ACL tear are associated with
improved success rates of meniscal repair. Meniscus tears on the contralateral side of the knee
and articular cartilage defects are not associated with improved healing rates.

RESPONSES FOR QUESTIONS 5 THROUGH 6


1- Physical therapy and a home exercise program
2- Corticosteroid injection
3- Arthroscopic debridement
4- Microfracture
5- Osteochondral autograft transplantation (OAT)
Match the treatment above with the clinical scenario below

Question 5 of 100
A 16-year-old female basketball player has a 4-week history of anterior knee pain during
practice. She also notes pain while sitting at her desk in class and pain while going down stairs.
An examination reveals mild patellofemoral crepitus with no patellar apprehension and normal
patellar tracking. Radiographs are unremarkable. MR imaging reveals no chondral or
osteochondral lesions of the patella or trochlear groove.
1- Physical therapy and a home exercise program
2- Corticosteroid injection
3- Arthroscopic debridement
4- Microfracture
5- Osteochondral autograft transplantation (OAT)

4
AAOS 2016 Sports medicine

PREFERRED RESPONSE: 1- Physical therapy and a home exercise program

Question 6 of 100
A 20-year-old college soccer player comes for an evaluation 6 months after an injury
during which he landed awkwardly from a jump. Although physical therapy, ice, and activity
modification have helped him return to baseline motion, strength, and swelling, he continues
to have lateral knee pain. He also notes a popping sensation on the lateral side of his knee with
activity. A Lachman test, anterior and posterior drawer tests, a pivot shift test, and McMurray
test findings are all negative. MR images reveal a 12-mm x 15-mm osteochondral defect in the
lateral femoral condyle with full-thickness cartilage loss and approximately 4 mm of
subchondral bone loss.
1- Physical therapy and a home exercise program
2- Corticosteroid injection
3- Arthroscopic debridement
4- Microfracture
5- Osteochondral autograft transplantation (OAT)

PREFERRED RESPONSE: 5- Osteochondral autograft transplantation (OAT)

DISCUSSION
Patellofemoral pain in a young athlete without patellar instability or a chondral or
osteochondral defect often can be managed with nonsurgical treatment such as physical
therapy and a home exercise program. Microfracture surgery is associated with good short-
term results for younger athletes. Patients with no history of prior surgery, primary chondral
rather than osteochondral lesions, and lesions smaller than 2 cm have experienced the best
results. Microfracture surgery performed for chondral lesions of the central aspect of the
medial femoral condyle is associated with worse results. Decreased activity levels over time
of patients who undergo microfracture surgery are a concern. OAT provides good outcomes
and return-to-sports rates for athletic people who are younger and have lesions smaller than 2
cm. Patients with lesions on the lateral femoral condyle have better success rates. Both
microfracture surgery and OAT provide better results for chondral defects than osteochondral
defects. OAT is associated with better results than microfracture for medium-sized lesions
between 2 cm and 4 cm, while autologous chondrocyte implantation yields better improvement
for patients with defects larger than 4 cm. All of the surgical techniques listed for articular
cartilage repair are associated with better outcomes for patients younger than age 30.

5
AAOS 2016 Sports medicine

Figure 7

Question 7 of 100
Figure 7 is the MR image of a 43-year-old man who has left shoulder pain with a traumatic
rotator cuff tear after a fall. An examination reveals active forward elevation at 120 degrees
and positive Yergason and lift-off test results. Arthroscopy reveals that the articular surfaces
of the glenohumeral joint have a normal appearance without significant degenerative changes.
What is the most appropriate treatment at this time?
1- Rotator cuff repair and biceps tenodesis
2- Rotator cuff repair and loose body removal
3- Latissimus dorsi transfer
4- Bankart repair

PREFERRED RESPONSE: 1- Rotator cuff repair and biceps tenodesis

DISCUSSION

6
AAOS 2016 Sports medicine

Video 7 for reference

The MR image shows medial subluxation of the biceps tendon, which can be confused
with an articular loose body. In the clinical scenario of biceps instability/subluxation, the
rationale regarding tenodesis is to address the painful dislocation and subluxation of the biceps
tendon from the bicipital groove.
The MR image does not show a loose body or Bankart lesion. Patients with irreparable
rotator cuff tears with a severe external rotation deficit and a deficient teres minor may
experience a better functional result with latissimus dorsi transfer.

Question 8 of 100
Augmentation of a Brostrm repair with the mobilized lateral portion of the extensor
retinaculum (Gould modification) is expected to produce
1- higher risk for iatrogenic nerve injury.
2- decreased ankle range of motion 6 weeks after surgery.
3- no significant biomechanical difference in initial ankle stability.
4- a significantly lower incidence of osteoarthritis on long-term follow-up.

PREFERRED RESPONSE: 3- no significant biomechanical difference in initial ankle


stability.

DISCUSSION

7
AAOS 2016 Sports medicine

Multiple biomechanical studies have investigated the contribution of the Gould


modification with the Brostrm anatomic repair for chronic ankle instability. No studies to
date have demonstrated a statistically significant difference in initial ankle stability with
inclusion of the Gould modification or augmentation of the repair with a mobilized lateral
portion of the extensor retinaculum. No clear association exists between the Brostrm-Gould
repair technique and risk for nerve injury, postsurgical range of motion, or incidence of
osteoarthritis on long-term follow-up.

Figure 9

CLINICAL SITUATION FOR QUESTIONS 9 THROUGH 13


Figure 9 is the MR image of a 36-year-old athlete who is tackled from behind and falls
forward onto his left knee. He has pain, swelling, and stiffness. Examination includes a
moderate effusion, positive quadriceps active test, and normal Lachman test finding.

Question 9 of 100
The injured structure is composed of an
1- anterolateral bundle that is tight in flexion and a posteromedial bundle that is tight in
extension.
2- anterolateral bundle that is tight in extension and a posteromedial bundle that is tight in
flexion.

8
AAOS 2016 Sports medicine

3- anteromedial bundle that is tight in flexion and a posterolateral bundle that is tight in
extension.
4- anteromedial bundle that is tight in extension and a posterolateral bundle that is tight in
flexion.

PREFERRED RESPONSE: 1- anterolateral bundle that is tight in flexion and a


posteromedial bundle that is tight in extension.

Question 10 of 100
Left untreated, injury to this structure most likely will lead to degenerative changes in
1- medial and lateral compartments.
2- medial and patellofemoral compartments.
3- lateral and patellofemoral compartments.
4- the patellofemoral compartment only.

PREFERRED RESPONSE: 2- medial and patellofemoral compartments.

Question 11 of 100
If the patient chooses surgical reconstruction, he should be advised that, when compared
to a transtibial technique, the tibial inlay technique has been shown to provide
1- stronger initial graft fixation.
2- more anatomic positioning of tibial fixation.
3- more natural knee kinematics during deep flexion.
4- more graft protection during cyclic loading.

PREFERRED RESPONSE: 4- more graft protection during cyclic loading.

Question 12 of 100

9
AAOS 2016 Sports medicine

This patient elects nonsurgical treatment and later experiences persistent instability.
Examination reveals an asymmetric Dial test finding and a varus thrust during ambulation.
Which osteotomy and correction appropriately addresses this chronic instability pattern?
1- Distal femoral/opening lateral wedge osteotomy
2- Distal femoral/closing lateral wedge osteotomy
3- High tibial osteotomy; opening medial wedge with increased tibial slope
4- High tibial osteotomy; closing lateral wedge with decreased tibial slope

YOUR RESPONSE: 4- High tibial osteotomy; closing lateral wedge with decreased tibial
slope

Question 13 of 100
Risk for vascular injury during transtibial drilling for reconstruction of this injury is
increased by
1- accessory incisions.
2- use of tapered drill bits.
3- use of oscillating drills.
4- greater knee extension.

PREFERRED RESPONSE: 4- greater knee extension.

DISCUSSION
The clinical description and MR image point to an injury to the posterior cruciate ligament
(PCL). This ligament is thought to be primarily composed of anterolateral and posteromedial
bundles, with the former tightening in flexion and the latter in extension. Because of alterations
in knee kinematics and increased varus alignment in PCL insufficiency, contact stresses and
cartilage loads increase in the patellofemoral and medial compartments. Although good
outcomes may be obtained with transtibial, open inlay, and arthroscopic inlay techniques, 1
major difference is the creation of the killer-turn during the transtibial approach. This sharp
turn in the graft as it emerges from the tibia appears to lead to more pronounced attenuation
and thinning of the graft during cyclic loading.
In Question 12, the scenario describes a patient with chronic PCL and posterolateral corner
(PLC) injury, as evidenced by the varus thrust and abnormal Dial test finding. A valgus-

10
AAOS 2016 Sports medicine

producing osteotomy may be effective, and, in fact, may be the only treatment necessary to
address chronic PLC injury. Accordingly, an opening lateral osteotomy would not be
appropriate. Of the remaining responses, an osteotomy that increases tibial slope would also
address the PCL deficiency by reducing posterior tibial sag.
Vascular injury is an uncommon, but potentially devastating, complication associated with
PCL surgery and may occur regardless of the technique used. Numerous strategies have been
described to reduce the risk, including use of a posteromedial accessory incision to allow finger
retraction of the popliteal neurovascular bundle, oscillating drills to prevent excessive soft-
tissue entanglement, and tapered (rather than square) drill bits that may minimize cut-out of
sharp edges as drilling reaches the posterior tibial cortex. Knee extension lessens, rather than
increases, the distance between the posterior tibia and the neurovascular bundle and increases,
not lessens, risk for vascular injury.

Question 14 of 100
A patient underwent a right hip arthroscopy, CAM resection, and labral repair while
positioned supine on a fracture table with a perineal post. The leg was in traction for 4 hours,
and no intrasurgical complications were noted. At the 2-week follow-up appointment, the
patient was experiencing numbness and tingling in the perineum on the surgical side and noted
pain predominantly while sitting. What is the likely cause of these symptoms?
1- Traction injury to the sciatic nerve
2- Traction injury to the femoral nerve
3- Compression injury to the pudendal nerve
4- Direct injury to the lateral femoral cutaneous nerve

PREFERRED RESPONSE: 3- Compression injury to the pudendal nerve

DISCUSSION
Although all of these responses are known complications related to hip arthroscopy, the
symptoms of perineal numbness and pain associated with prolonged traction time indicate a
compression injury to the pudendal nerve against the perineal post used to provide counter
traction. Perineal numbness usually occurs on the surgical side, with pain in the area of the
anus to the penis/clitoris. Pain is predominantly experienced while sitting, but is relieved when
sitting on a toilet. Pain can be relieved with a diagnostic pudendal nerve block. This injury is
not unique to hip arthroscopy; it also is described in the trauma literature. To prevent
compression-type injuries, a well-padded post larger than 9 cm in diameter should be
positioned against the medial thigh. Traction force should be kept to a minimum and the

11
AAOS 2016 Sports medicine

extremity positioned in slight abduction. Continuous traction time should not exceed 2 hours,
with intermittent traction used during prolonged procedures.

Figure 15a Figure 15b

Question 15 of 100
Figures 15a and 15b are intrasurgical photographs from the posterolateral viewing portal
that were taken at the beginning and end of a right shoulder arthroscopic procedure performed
on a 54-year-old man. This technique demonstrates superior results compared to traditional
arthroscopic techniques when evaluating which outcome?
1- Time to healing
2- Retear rate
3- Functional outcome scores
4- Postsurgical pain scores

PREFERRED RESPONSE: 2- Retear rate

DISCUSSION
The images reveal a medium-sized tear of the rotator cuff. As more clinical studies are
published comparing double-row with single-row rotator cuff repair, it has become clear that
the retear rate is lower with a double-row construct for small and medium-sized tears. This
may be attributable to the stronger time-zero repair construct that double-row repair provides.
No study to date has demonstrated a significant difference in clinical outcomes (functional and
pain scores at any time) or time to healing between the 2 techniques.

12
AAOS 2016 Sports medicine

Figure 16

CLINICAL SITUATION FOR QUESTIONS 16 THROUGH 17


Figure 16 is the MR image of a 14-year-old football player who injured his right knee
during a game. He describes feeling a "pop," and then he needed help walking off the field.
His knee is visibly swollen.

Question 16 of 100
Knee range of motion is between 0 degrees and 70 degrees. What is the most appropriate
treatment option?
1- Open reduction and internal fixation of the lateral condyle
2- Microfracture of the chondral defect
3- Immediate anterior cruciate ligament (ACL) reconstruction
4- Delayed ACL reconstruction

PREFERRED RESPONSE: 4- Delayed ACL reconstruction

Question 17 of 100

13
AAOS 2016 Sports medicine

The patient has no postsurgical complications and begins physical therapy rehabilitation.
The boy and his parents stress they want to get the therapy over with as fast as possible to
expedite his return to sports, and the surgeon and rehabilitation team consider their request.
Compared to nonaccelerated rehabilitation, patients who follow an early accelerated
rehabilitation protocol experience
1- increased laxity.
2- increased risk for graft failure.
3- no differences in long-term results.
4- lower Knee Injury and Osteoarthritis Outcome Scores (KOOS).

PREFERRED RESPONSE: 3- no differences in long-term results.

DISCUSSION
The MR image shows bone bruises (kissing contusions) consistent with an ACL tear.
During the ACL subluxation event, the posterolateral tibia plateau subluxes anteriorly, making
contact with the mid portion of the lateral femoral condyle and resulting in this characteristic
bone bruise pattern on MRI. Randomized clinical trials comparing early accelerated vs
nonaccelerated rehabilitation programs have demonstrated no significant differences in long-
term results with regard to function, reinjury, and successful return to play. These studies did
not address timing of return to play with an early accelerated rehabilitation program. At 2 and
3 years postsurgically, there are no differences in laxity, number of graft failures, or KOOS
scores.

Question 18 of 100
A 32-year-old volleyball player has dull posterior shoulder pain. An examination reveals
moderate external rotation weakness with his arm at his side, but normal strength on
supraspinatus isolation. Deltoid and supraspinatus bulk appear normal, although there appears
to be mild infraspinatus atrophy. Sensation is normal throughout the shoulder and shoulder
girdle. What is the most likely diagnosis?
1- Calcified transverse scapular ligament
2- Parsonage-Turner syndrome
3- Spinoglenoid notch cyst
4- Quadrilateral space syndrome

14
AAOS 2016 Sports medicine

PREFERRED RESPONSE: 3- Spinoglenoid notch cyst

DISCUSSION
This clinical scenario describes a patient with an isolated injury affecting the infraspinatus
muscle. The anatomic location of such a lesion would be at the spinoglenoid notch, at which
the suprascapular nerve may be compressed distal to its innervation of the supraspinatus but
proximal to the infraspinatus innervation. A calcified transverse scapular ligament would also
affect the suprascapular nerve but is proximal to the innervation of both muscles. Quadrilateral
space syndrome would affect innervation of the deltoid (and teres minor). Parsonage-Turner
syndrome is a more diffuse, and often severely painful, brachial plexus neuropathy.

Figure 19a Figure 19b

Question 19 of 100
A 25-year-old woman has lower leg pain during exercise without numbness, tingling, or
weakness. The symptoms resolve by the following day. Compartment pressure measurements
obtained 1 minute after exercise are shown in Figure 19a (Table 1). She undergoes anterior
compartment fasciotomy with complete resolution of symptoms. Two years later, she has
recurrent pain and tightness with exercise. Radiographs, a technetium bone scan, and
noninvasive vascular study findings are normal. Compartment pressure measurements
obtained 1 minute after exercise are shown in Figure 19b (Table 2). What is the most likely
etiology for her recurrent symptoms?
1- Misdiagnosis
2- Hematoma formation
3- Postsurgical fibrosis
4- Failure to recognize involvement of other compartments

15
AAOS 2016 Sports medicine

PREFERRED RESPONSE: 3- Postsurgical fibrosis

DISCUSSION
Exertional compartment syndrome involves an increase in compartment pressure caused
by exercise or sports activity that restricts blood flow in the compartment, resulting in pain
with continued activity. Compartment pressures of at least 15 mm Hg measured at rest, at least
30 mm Hg measured 1 minute after exercise, and at least 20 mm Hg measured 5 minutes after
exercise are diagnostic. Surgical fasciotomy for exertional compartment syndrome is
successful for the majority of patients, but recurrence rates as high as 20% have been reported.
Scar formation within the fascial defect can result in recurrent symptoms and/or nerve
entrapment, and recurrence is typically observed after an initial symptom-free period. In a
series of 18 patients, recurrent symptoms occurred at a mean of 23.5 months after the index
procedure. Other potential causes of recurrence include inadequate fascial release, failure to
recognize involvement of other compartments, nerve compression, and misdiagnosis. Surgical
complications after fasciotomy include hemorrhage leading to excessive fibrosis,
neurovascular injury, and hematoma or seroma formation.

Question 20 of 100
A 16-year-old swimmer has right shoulder pain with activity. She describes the continued
sensation that her shoulder is loose. She has been in physical therapy for 7 months to work
on strengthening the muscles around her shoulder and scapula. She denies being able to
voluntarily dislocate her shoulder. Upon examination, you can feel the humeral head slide over
the glenoid rim both anteriorly and posteriorly with the load and shift test. She has a grade III
sulcus sign. What is the most appropriate next step?
1- Arthroscopic superior labrum anterior to superior repair
2- Arthroscopic Bankart repair
3- Latarjet procedure
4- Capsulorrhaphy

PREFERRED RESPONSE: 4- Capsulorrhaphy

DISCUSSION
Nonsurgical treatment with activity modification and physical therapy is generally
considered the first-line approach for young athletes with multidirectional instability (MDI) of
the shoulder. Physical therapy focuses on exercises to strengthen the scapular stabilizers and
rotator cuff muscles and restore scapulohumeral rhythm. Although a definitive length of time

16
AAOS 2016 Sports medicine

to assess physical therapy failure is not known, many surgeons believe that a patient with MDI
should undergo at least 6 months of physical therapy and activity modification before
considering surgery. Although an open inferior capsular shift has historically been considered
the gold standard for surgical treatment for MDI, studies have shown good success rates for
arthroscopic capsulorrhaphy. Arthroscopy can allow a surgeon to assess all intra-articular
structures and address a patients particular problem based on arthroscopic findings.

Figure 24a Figure 24b

CLINICAL SITUATION FOR QUESTIONS 21 THROUGH 25


A 19-year-old collegiate offensive lineman injures his left elbow in a scrimmage. He
reports reaching out with his left arm to prevent the defensive player from getting around him,
and, as he grabbed the player, his elbow was forced into extension. He had immediate pain
and weakness and heard a pop. He has mild swelling in the antecubital fossa and a
prominent-appearing biceps muscle belly. His hook test result is abnormal at the elbow.

Question 21 of 100

17
AAOS 2016 Sports medicine

Which type of contraction of the involved muscle most likely resulted in this lineman's
injury?
1- Eccentric
2- Concentric
3- Isometric
4- Isokinetic

PREFERRED RESPONSE: 1- Eccentric

Question 22 of 100
The most substantial functional deficit that may develop if no surgical treatment is
provided is
1- elbow flexion strength.
2- elbow supination strength.
3- lack of terminal extension at the elbow.
4- decrease of elbow pronation strength.

PREFERRED RESPONSE: 2- elbow supination strength.

Question 23 of 100
The athlete decides to undergo surgery. Which complication is most commonly associated
with surgical repair of this injury?
1- Posterior interosseous nerve (PIN) palsy
2- Infection
3- Lateral antebrachial cutaneous neuropraxia (LABCN)
4- Symptomatic heterotopic ossification

PREFERRED RESPONSE: 3- Lateral antebrachial cutaneous neuropraxia (LABCN)

18
AAOS 2016 Sports medicine

Question 24 of 100
The athlete undergoes repair of the injury, and postsurgical radiographs are shown in
Figures 24a and 24b. At his first postsurgical visit he reports no pain but describes weakness
in his hand and decreased sensation over his lateral forearm. Upon examination, he has
decreased 2-point discrimination over the lateral forearm and an inability to actively extend
his thumb and fingers at the metacarpophalangeal joints. He can extend at the finger
interphalangeal joints. He can extend his wrist weakly, and it deviates radially as he extends.
His distal sensation is intact. Considering his examination findings, which 2 nerves are
injured?
1- PIN and radial nerve
2- PIN and LABCN
3- Median nerve and LABCN
4- Radial nerve and LABCN

PREFERRED RESPONSE: 2- PIN and LABCN

Question 25 of 100
After the athlete undergoes the appropriate treatment of the postsurgical complication and
recovers without further incident, which muscle most likely will be last to experience return
of function?
1- Extensor indicis proprius (EIP)
2- Extensor digiti quinti (EDQ)
3- Extensor digitorum communis (EDC)
4- Extensor carpi ulnaris (ECU)

PREFERRED RESPONSE: 1- Extensor indicis proprius (EIP)

DISCUSSION
This patient sustained an eccentric contracture (muscle lengthening while contracting) of
his biceps muscle while trying to stop a defender from getting around him. This in turn caused
failure of the distal biceps tendon, as evidenced by pain in the antecubital fossa, lack of elbow
supination strength, and his positive biceps active test finding (supination/pronation of the
forearm showing no motion of the biceps muscle belly). Eccentric contractors have the highest

19
AAOS 2016 Sports medicine

potential for building strength but also are at highest risk for injury. Concentric (muscle
shortening with contraction), isometric (no change in muscle length with contracture), and
isokinetic (constant velocity of muscle contraction with a variable force) do not describe the
mechanism detailed.
The loss of distal biceps attachment will result in loss of elbow supination strength in
flexion (the biceps is the only supinator to cross the elbow) while still retaining elbow flexion
(albeit weakened) because of the other elbow flexors (brachioradialis and brachialis).
Consequently, treatment should be anatomic repair of the distal biceps insertion, which can be
performed with a 2-incision or 1-incision technique. Although all of the listed complications
have been reported with these techniques, LABC neuropraxia is by far the most common.
Radiographs show that this athletes injury was repaired using a 1-incision technique with a
cortical fixation device and a radial bone tunnel. This technique has gained favor because of
its decreased incidence of heterotopic ossification and radioulnar synostosis compared to the
2-incision technique. The most troubling complication for most surgeons is the development
of a PIN palsy, which this patient clearly demonstrates in addition to the more common
LABCN upon postsurgical examination. Because the LABC nerve injury is typically a
neuropraxia from retraction, a period of observation is indicated. PIN injury can result from
excessive traction during surgical exposure or from entrapment by the fixation button.
Considering the anatomy of the PIN, successful recovery of the nerve typically progresses
based on the distance from the origin of the nerve to the muscle indicated. The EIP is the most
distal muscle innervated and can be expected to recover last. First to return would be the EDC
followed by the ECU, EDQ, and, finally, the EIP.

RESPONSES FOR QUESTIONS 26 THROUGH 27


1- Anterior tibial artery
2- Posterior tibial artery
3- Superficial peroneal nerve
4- Deep peroneal nerve
Match the neurovascular structure at risk (listed above) with the compartment undergoing
fasciotomy (listed below).

Question 26 of 100
Lateral compartment
1- Anterior tibial artery
2- Posterior tibial artery
3- Superficial peroneal nerve
4- Deep peroneal nerve

20
AAOS 2016 Sports medicine

PREFERRED RESPONSE: 3- Superficial peroneal nerve

Question 27 of 100
Deep posterior compartment
1- Anterior tibial artery
2- Posterior tibial artery
3- Superficial peroneal nerve
4- Deep peroneal nerve

PREFERRED RESPONSE: 2- Posterior tibial artery

DISCUSSION
The structures at risk are the anterior tibial artery and deep peroneal nerve in the anterior
compartment, superficial peroneal nerve in the lateral compartment, sural nerve in the
superficial posterior compartment, and posterior tibial nerve and posterior tibial and peroneal
arteries and veins in the deep posterior compartment.

Figure 28a Figure 28b

21
AAOS 2016 Sports medicine

Question 28 of 100
Figures 28a and 28b are the MR images of a 30-year-old man who has right shoulder pain
and difficulty throwing a football. His history includes a shoulder injury from a skiing accident
2 years ago. He has not had a recent shoulder injury. Which shoulder motion is most likely to
demonstrate weakness?
1- Shoulder abduction
2- Shoulder internal rotation
3- Shoulder external rotation
4- Shoulder adduction

PREFERRED RESPONSE: 3- Shoulder external rotation

DISCUSSION
The MR images reveal a large paralabral cyst extending into the spinoglenoid notch. This
cyst can be expected to compress the branch of the suprascapular nerve to the infraspinatus.
Compression of this branch could lead to weakness in the infraspinatus, which would manifest
as external rotation weakness. Shoulder abduction would be unaffected because the axillary
and main suprascapular nerves would be intact. Shoulder internal rotation and adduction would
be unaffected because the subscapularis and pectoralis would be unaffected.

Figure 29

22
AAOS 2016 Sports medicine

Question 29 of 100
A 24-year-old collegiate pitcher has had increasing pain over his medial elbow for 3
months. He has point tenderness over his medial epicondyle and reproduction of his symptoms
with a valgus stress test. Which phase of the throwing cycle shown in Figure 29 will most
likely reproduce his symptoms?
1- B
2- C
3- D
4- E

PREFERRED RESPONSE: 2- C

DISCUSSION
This patient is experiencing soreness over his medial (ulnar) collateral ligament. Valgus
overload is likely to reproduce his symptoms and is most pronounced during the late cocking
phase of the throwing cycle. In windup, very little elbow torque is required. In early cocking,
the arm is getting loaded, and maximum valgus is not yet achieved at the elbow. In acceleration
and deceleration, more force is generated at the level of the shoulder joint.

CLINICAL SITUATION FOR QUESTIONS 30 THROUGH 33


A 42-year-old man has a 4-month history of right shoulder pain. He denies any specific
injury that initiated his symptoms. He says that his pain is worse when reaching overhead or
lifting an object across his body. He describes his pain as being located along the front of his
shoulder, although he occasionally feels the pain on the side of his neck. Upon examination,
you find mild tenderness to palpation at his acromioclavicular (AC) joint. Radiographs show
osteophytes and narrowing of the AC joint.

Question 30 of 100
Which examination test is most specific for pain related to AC joint osteoarthritis?
1- Pain when the shoulder is brought into 90 degrees of forward flexion and maximal
adduction across the body
2- Pain when the shoulder is brought into 90 degrees of forward flexion while you
internally rotate the shoulder
3- Pain when the shoulder is brought into forward elevation while your hand stabilizes the
scapula

23
AAOS 2016 Sports medicine

4- Pain when the patient resists downward pressure while the arm is in 90 degrees of
abduction and in the scapular plane

PREFERRED RESPONSE: 1- Pain when the shoulder is brought into 90 degrees of


forward flexion and maximal adduction across the body

Question 31 of 100
The patient inquires about nonsurgical treatment for his AC joint arthritis. You should
advise him that
1- physical therapy is of little use because of his substantial degenerative changes.
2- a corticosteroid injection may help to control symptoms, although access to the joint is
challenging.
3- 4 weeks of sling immobilization will likely provide the best long-term relief of
symptoms.
4- he should strongly consider surgical treatment because nonsurgical options rarely
provide relief.

PREFERRED RESPONSE: 2- a corticosteroid injection may help to control symptoms,


although access to the joint is challenging.

Question 32 of 100
What is the most common complication associated with distal clavicle resection for AC
joint osteoarthritis?
1- AC joint instability
2- Persistent pain
3- Nerve injury
4- Arthritis recurrence

PREFERRED RESPONSE: 2- Persistent pain

Question 33 of 100

24
AAOS 2016 Sports medicine

After performing an open distal clavicle excision and resecting 15 mm of distal clavicle,
which potential concern for shoulder function could result?
1- Persistent pain attributable to inadequate resection
2- Complex regional pain syndrome
3- Fracture
4- AC joint instability

PREFERRED RESPONSE: 4- AC joint instability

DISCUSSION
AC joint arthritis often is marked by pain along the anterior and superior aspects of the
shoulder. It can occasionally radiate into the trapezius and the anterolateral neck region. A
patient may have tenderness to palpation directly at the AC joint or pain with the cross-body
adduction stress test and the O'Brien active compression test. During the cross-body adduction
test, this patient has pain when the examiner lifts his arm in 90 degrees of forward flexion and
maximally adducts it across his body. Although the cross-body adduction test is the most
sensitive provocative test for AC joint osteoarthritis at 77%, the OBrien active compression
test has been shown to be most specific at 95%.
Physical therapy, rest, activity modification, and other nonsurgical treatments might not
reverse osteoarthritis changes at the AC joint, but these interventions can often help improve
pain, range of motion, and function. A corticosteroid injection into the AC joint may be an
option if nonsurgical treatments do not work, although Wasserman and associates
demonstrated that only 44% of AC joint injections accurately entered the joint.
Persistent pain is the most common complication following distal clavicle excision.
Although the exact amount of distal clavicle that should be resected is a topic of debate,
resection of 10 mm or more of the distal clavicle may lead to instability of the AC joint,
especially if the AC capsule is sectioned.

25
AAOS 2016 Sports medicine

Figure 34

Question 34 of 100
making a cut toward the ball. He felt a pop and his leg gave way. During physical
examination, as the knee is moved from full extension into flexion with an internal rotation
and valgus force, you notice a clunk within the knee. What is the most likely biomechanical
basis for the clunk?
1- In extension with internal rotation/valgus force, the medial tibial plateau is subluxated;
with flexion, the medial tibial plateau reduces
2- In extension with internal rotation/valgus force, the medial tibial plateau is reduced;
with flexion, the medial tibial plateau subluxates
3- In extension with internal rotation/valgus force, the lateral tibial plateau is reduced; with
flexion, the lateral plateau subluxates
4- In extension with internal rotation/valgus force, the lateral tibial plateau is subluxated;
with flexion, the lateral plateau reduces

PREFERRED RESPONSE: 4- In extension with internal rotation/valgus force, the lateral


tibial plateau is subluxated; with flexion, the lateral plateau reduces

DISCUSSION

26
AAOS 2016 Sports medicine

This patient sustained an isolated anterior cruciate ligament (ACL) injury based upon the
mechanism described and examination findings. The finding that produces the clunk is the
pivot-shift maneuver, which is positive in a knee with an incompetent ACL. With an ACL-
deficient knee in full extension and internal rotation, the lateral tibial plateau subluxates
anteriorly. As the knee is flexed, the lateral tibial plateau slides posteriorly into a reduced
position, causing an audible clunk. Response 4 correctly describes the pathomechanics that
result in the audible clunk heard during the pivot-shift maneuver. Responses 1 and 2 are
incorrect because they describe the medial tibial plateau, which is not part of the
pathomechanics of the pivot shift. Response 3 is incorrect because in extension, the lateral
tibial plateau is subluxated, not reduced.

Figure 35

Question 35 of 100
Figure 35 is the MR image of an 18-year-old man who has had knee pain with running for
5 months. What is the most appropriate treatment?
1- Arthroscopic or open reduction and internal fixation with possible bone grafting
2- Arthroscopic chondroplasty
3- No weight-bearing activity for 6 weeks and then re-evaluate
4- Retrograde subchondral drilling without fixation

27
AAOS 2016 Sports medicine

PREFERRED RESPONSE: 1- Arthroscopic or open reduction and internal fixation with


possible bone grafting

DISCUSSION
The MR image shows an osteochondritis dissecans (OCD), which is an acquired lesion of
the subchondral bone. Patients with OCD initially report nonspecific pain and variable
amounts of swelling. Initial radiographs help to identify the lesion and establish the physes
status. MRI is useful for assessing potential for the lesion to heal with nonsurgical treatment.
This lesion is unstable, considering the fluid line between the OCD and the underlying normal
bone. Nonsurgical treatment is appropriate for small, stable lesions in patients with open
physes and focuses on activity restriction for 3 to 9 months. Surgical treatment is necessary to
address unstable or detached lesions. Stable lesions with intact articular cartilage can be treated
with subchondral drilling to stimulate vascular ingrowth, with radiographic healing at an
average of 4.4 months. Fixation is indicated for unstable or hinged lesions, and stabilization
of the fragment can be achieved using a variety of implants through an arthroscopic or open
approach. The fragment should be salvaged and the normal articular surface restored whenever
possible.

Question 36 of 100
A 17-year-old male soccer player sustains repeated lateral patellar dislocations refractory
to physical therapy, bracing, and taping. After a workup including radiographs and MRI, the
orthopaedic surgeon considers an isolated tibial tubercle osteotomy (TTO). He plans a 60-
degree anteromedialization to address instability and to unload the patellofemoral joint. What
is a relative contraindication to this procedure?
1- Grade III chondrosis of the proximal patella
2- Caton-Deschamps ratio of 1:1
3- Tibial tubercle-trochlear groove (TT-TG) distance of 21 mm
4- Q angle of 17 degrees

PREFERRED RESPONSE: 1- Grade III chondrosis of the proximal patella

DISCUSSION
TTO is a common treatment for patellofemoral instability. The angle of correction must
be customized to each patients anatomy. For this patient, the orthopaedic surgeon plans an
osteotomy that will both anteriorize and medialize the tubercle. This will consistently result in

28
AAOS 2016 Sports medicine

a change of patellofemoral kinematics and contact pressures. Medialization decreases lateral


and increases medial patellofemoral contact pressures, and anteriorization shifts contact
pressures from distal to proximal. Significant anteriorization may not be desired in a patient
with proximal patellar chondrosis unless a concomitant chondral procedure is performed as
well. The patellar height (Caton-Deschamps ratio) is normal, precluding the need for
distalization but not medialization. The TT-TG distance, at more than 20 mm, is a strong
indication for osteotomy. The Q angle, although a less precise indicator of malalignment, is
also elevated and would be considered an indication for osteotomy.

Figure 39a Figure 39b

29
AAOS 2016 Sports medicine

Figure 39c Figure 39d

30
AAOS 2016 Sports medicine
Figure 40a Figure 40b

CLINICAL SITUATION FOR QUESTIONS 37 THROUGH 41


A healthy, active 18-year-old man has acute-onset right knee pain and an inability to fully
extend his knee following an attempt to stand from a seated position yesterday. He sustained
a noncontact injury to his right knee while playing basketball 2 years ago and underwent
primary anterior cruciate ligament (ACL) reconstruction with bone-patella-tendon-bone
autograft and medial meniscus repair. He sustained another noncontact injury to the same knee
8 months later and underwent a revision ACL reconstruction using soft-tissue allograft and
revision medial meniscus repair. He reports multiple episodes of giving way of his knee, but
no pain prior to yesterdays acute injury.

Question 37 of 100
Which risk factor is associated with the highest rate of recurrent ACL rupture following
reconstruction?
1- Family history of ACL rupture
2- Patient age
3- Type of graft used
4- A supervised accelerated rehabilitation program

PREFERRED RESPONSE: 2- Patient age

Question 38 of 100
The patient underwent a primary meniscal repair using a second-generation all-inside
meniscal repair system. In the setting of concomitant ACL reconstruction, which medial
meniscus repair healing rate is appropriate to quote when use of these devices is discussed?
1- 45%
2- 55%
3- 65%
4- 85%

PREFERRED RESPONSE: 4- 85%

31
AAOS 2016 Sports medicine

Question 39 of 100
Figures 39a through 39d are this patients radiographs and sagittal MR images. An
examination reveals a painful knee with limited motion from 10 degrees shy of full extension
to 100 degrees of knee flexion. He has a positive Lachman test result and negative Dial test
result. A pivot shift cannot be performed because of his lack of motion. What is the best next
treatment step?
1- Arthroscopic revision ACL reconstruction
2- Arthroscopic revision medial meniscus repair
3- Arthroscopic debridement of a cyclops lesion
4- Arthroscopic-assisted revision ACL reconstruction with partial medial meniscectomy

PREFERRED RESPONSE: 4- Arthroscopic-assisted revision ACL reconstruction with


partial medial meniscectomy

Question 40 of 100
Presurgical radiographs and MR images are shown in Figures 39a through 39d. The patient
undergoes a second revision ACL reconstruction with a soft-tissue allograft. Postsurgical
radiographs are shown in Figures 40a and 40b. Assuming there is a well-tensioned graft in
each scenario, the new tunnel placement likely will result in improvement of which
examination maneuver?
1- Dial test
2- Pivot shift
3- Anterior drawer
4- Posterior drawer

PREFERRED RESPONSE: 2- Pivot shift

Question 41 of 100
The patient returns 4 days after surgery and says he has noticed a red, swollen knee since
yesterday. He reports a fever of 38.0C since last evening and denies traumatic injury. He has
an erythematous knee with a large, tense effusion; his range of motion is limited; and the
surgical incisions are not draining. Radiographs taken in the office show no change from the

32
AAOS 2016 Sports medicine

immediate postsurgical images. Aspiration in the office returns 50 cc of cloudy, blood-tinged


synovial fluid, and analysis of the fluid reveals a white blood cell count of 92000 (reference
range 4500-11000 /L). Which bacteria is most commonly responsible for this clinical
scenario?
1- Staphylococcus epidermidis
2- Staphylococcus aureus (S. aureus)
3- Propionibacterium acnes (P. acnes)
4- Beta-hemolytic Streptococcus

PREFERRED RESPONSE: 1- Staphylococcus epidermidis

DISCUSSION

Video 39 for reference

This patient has a history of failed primary and revision ACL reconstructions, both times
with medial meniscus repairs. The clinical scenario suggests a recurrent ACL injury with a
recurrent medial meniscus tear that is now locked. The most critical risk factor for ACL
reconstruction is age younger than 20 years. The meniscal repair success rate using an all-
inside device is between 80% and 90%. Traditionally, it was believed that healing rates were

33
AAOS 2016 Sports medicine

higher in ACL reconstruction, but current literature demonstrates a similar rate of healing
associated with ACL reconstruction and no reconstruction of stable knees.
The images show a vertical femoral tunnel resulting from this patients prior reconstruction
and revision. The MR images reveal a locked bucket-handle tear of the medial meniscus, and
the examination shows a positive Lachman test finding attributable to ACL graft failure. In
the setting of a young individual who has failed 2 meniscal repairs, a third repair is not
indicated. In addition to a revision ACL reconstruction to stabilize the knee, a partial medial
meniscectomy is indicated. An attempt at revision medial meniscus repair would be indicated
if the technique were poor in the first attempt, but a failed repair otherwise should indicate the
need for partial meniscectomy. The postsurgical images reveal a much more anatomic position
of the femoral tunnel that should provide better rotational control of the knee, thereby
improving the pivot shift (compared to the vertical femoral tunnel).
This patient has an obvious postsurgical infection based on the timing, examination, and
results of the aspiration. In multiple studies of septic arthritis following ACL reconstruction,
the most common pathogen was coagulase-negative staph (Staphylococcus epidermidis),
followed by S. aureus. If S. aureus is the causative pathogen, the rate of necessary graft
removal is higher because of the aggressive nature of this specific bacteria.

Question 42 of 100
Which sterilization method is expected to produce the most degradation of an allograft
used for anterior cruciate ligament reconstruction?
1- Deep freezing
2- Supercritical CO2 treatment
3- Gamma irradiation with 1.2 Mrad
4- Chlorhexidine gluconate 4% cleansing

PREFERRED RESPONSE: 2- Supercritical CO2 treatment

DISCUSSION
A biomechanical study compared unprocessed, irradiated (2.0 Mrad-2.8 Mrad), and
supercritical CO2-treated soft-tissue allografts and demonstrated a 27% to 36% decrease in
stiffness of the supercritical CO2-treated grafts. No significant difference was found between
the irradiated and untreated soft-tissue allografts. Low-dose (1.0 Mrad-1.2 Mrad) gamma
irradiation of bone-patellar-tendon-allograft has been shown to produce a 20% decrease in
graft stiffness. Deep freezing or cleansing with 4% chlorhexidine gluconate does not appear
to adversely affect the biomechanical properties of the allograft tissue.

34
AAOS 2016 Sports medicine

Figure 43a Figure 43b

CLINICAL SITUATION FOR QUESTIONS 43 THROUGH 46


Figures 43a and 43b are the MR arthrogram images of a 16-year-old, right-hand-dominant
baseball player who injured his left shoulder 4 weeks ago during a game. He now has pain,
weakness, and the inability to swing a bat and can no longer do push-ups. He denies prior
injury to his left shoulder. Radiographs are unremarkable.

Question 43 of 100
Which provocative maneuver is most likely to reproduce pain for this patient?
1- Forward flexion, abduction, and internal rotation
2- Forward flexion, abduction, and external rotation
3- Forward flexion, adduction, and internal rotation
4- Forward flexion, adduction, and external rotation

PREFERRED RESPONSE: 3- Forward flexion, adduction, and internal rotation

35
AAOS 2016 Sports medicine

Question 44 of 100
Which of the 4 muscles of the rotator cuff provides the most resistance to this patient's
direction of instability?
1- Subscapularis
2- Supraspinatus
3- Infraspinatus
4- Teres minor

PREFERRED RESPONSE: 1- Subscapularis

Question 45 of 100
The patient fails an extensive course of physical therapy and is unable to return to baseball.
He and his orthopaedic surgeon elect to proceed with surgery. During a repeat evaluation, he
has negative sulcus and Beighton sign findings, and radiographs show 5 degrees of glenoid
retroversion. What is the most appropriate surgical plan?
1- Arthroscopic infraspinatus tenodesis
2- Arthroscopic posterior labral repair
3- Arthroscopic capsular shift and rotator interval closure
4- Posterior glenoid opening-wedge osteotomy

PREFERRED RESPONSE: 2- Arthroscopic posterior labral repair

Question 46 of 100
What is the most likely complication after surgical treatment in this scenario?
1- Recurrent instability
2- Degenerative joint disease
3- Shoulder stiffness
4- Axillary nerve injury

PREFERRED RESPONSE: 3- Shoulder stiffness

36
AAOS 2016 Sports medicine

DISCUSSION
Posterior shoulder instability is a rare form of instability that often presents with pain rather
than feelings of instability. It often occurs in young athletes during activities that put the
shoulder in an at-risk position (flexion, adduction, and internal rotation). Repetitive
microtrauma can lead to posterior shoulder instability such as seen in football linemen.
Swinging a bat or golf club places the lead arm in a flexed, adducted, and internally rotated
position, which can lead to posterior translation of the humeral head that is forcibly reduced
in follow-through as seen in this patient.
The glenohumeral joint relies on static and dynamic stabilizers. Static stabilizers help
prevent instability at the end ranges of motion when the ligaments are taut. Dynamic stabilizers
work to prevent subluxation at midranges of motion at which the ligaments are lax. The rotator
cuff is integral as a dynamic stabilizer of the shoulder. It works through a process called
concavity compression. The 4 muscles of the rotator cuff compress the humeral head into the
concavity of the glenoid-labrum. This prevents the humeral head from subluxing during the
midranges of motion. Of the 4 rotator cuff muscles, the subscapularis is most important at
preventing posterior subluxation.
This patient has posterior instability, and various surgical techniques may be indicated
depending on findings. Arthroscopic labral repair is indicated for anterior instability.
Arthroscopic posterior labral repair is indicated for this patient because he has a posterior
labral tear and posterior instability. If a patient has ligamentous laxity (not seen in this scenario
because sulcus and Brighton sign findings would be negative), a posterior capsular shift with
rotator interval closure is indicated. If a patient has excessive glenoid retroversion, a posterior
opening-wedge osteotomy is appropriate.
The most common complication seen after arthroscopic posterior labral repair is stiffness,
followed by recurrent instability and degenerative joint disease.

Question 47 of 100
Which study is most useful for diagnosis of exertional compartment syndrome?
1- MRI
2- Arterial Doppler
3- Static compartment pressures
4- Exertional compartment pressures

PREFERRED RESPONSE: 4- Exertional compartment pressures

37
AAOS 2016 Sports medicine

DISCUSSION
The most sensitive study in the diagnosis of exertional compartment syndrome is
intracompartmental pressures taken at rest (compared to pressures taken immediately after
exercise). MRI often can reveal nonspecific muscle edema in exertional compartment
syndrome, but this is usually not diagnostic. Arterial Doppler studies are usually unremarkable
unless they are taken after exercise, in which case these findings may be abnormal.

Figure 48a Figure 48b

38
AAOS 2016 Sports medicine

Figure 48c Figure 48d

39
AAOS 2016 Sports medicine

Figure 48e Figure 48f

CLINICAL SITUATION FOR QUESTIONS 48 THROUGH 50


Figures 48a through 48f reveal the radiographs and MR images of a 30-year-old man who
has a 1-year history of atraumatic medial-sided left knee pain refractory to nonsurgical
measures.

Question 48 of 100
What is the most appropriate treatment?
1- Distal femoral varus osteotomy
2- Autologous chondrocyte implantation (ACI)
3- Fresh osteochondral allograft (OCA) transplantation
4- Arthroscopic microfracture

PREFERRED RESPONSE: 3- Fresh osteochondral allograft (OCA) transplantation

Question 49 of 100
In comparison to the lamina splendens, the deep zone of hyaline cartilage contains
1- larger-diameter collagen fibrils arranged perpendicular to the joint surface and a higher
concentration of proteoglycans.

40
AAOS 2016 Sports medicine

2- larger-diameter collagen fibrils arranged parallel to the joint surface and a higher
concentration of proteoglycans.
3- smaller-diameter collagen fibrils arranged perpendicular to the joint surface and a lower
concentration of proteoglycans.
4- smaller-diameter collagen fibrils arranged parallel to the joint surface and a lower
concentration of proteoglycans.

PREFERRED RESPONSE: 1- larger-diameter collagen fibrils arranged perpendicular to


the joint surface and a higher concentration of proteoglycans.

Question 50 of 100
Which factor has the most negative influence on the success of knee osteochondral
allograft transplantation?
1- Mechanical axis malalignment
2- Blood-type matching of the host and donor
3- Sex of the donor
4- Failed previous cartilage procedure

PREFERRED RESPONSE: 1- Mechanical axis malalignment

DISCUSSION
The images illustrate a large unstable osteochondral lesion of the medial femoral condyle.
Radiographs and MR images clearly show deep subchondral bone involvement. The
appropriate choice of surgery is OCA transplantation, which is indicated for primary treatment
of large cartilage lesions, osteochondral lesions, and salvage procedure from failed prior
cartilage surgery. Correction of mechanical axis malalignment, ligamentous insufficiency, and
meniscal deficiency should also be addressed. ACI alone or an arthroscopic microfracture
procedure would not address the bone defect, leaving an uneven articular surface. Although
an osteotomy may be a viable choice, a distal femoral varus osteotomy would increase the
contact pressure in the medial compartment and worsen the situation.
The histologic anatomy of articular cartilage is well described. The superficial layer or
lamina splendens contains a small amount of proteoglycan with collagen fibrils arranged
parallel to the articular surface. In contrast, the deep zone contains the largest-diameter

41
AAOS 2016 Sports medicine

collagen fibrils, oriented perpendicular to the joint surface, and the highest concentration of
proteoglycans.

Figure 51a Figure 51b

42
AAOS 2016 Sports medicine

Video 54 this video is uploaded at

CLINICAL SITUATION FOR QUESTIONS 51 THROUGH 54


Figures 51a and 51b are the radiographs of an 18-year-old football linebacker who was
involved in a tackle and fell onto an outstretched left arm. He had immediate pain and
deformity of his left elbow.

Question 51 of 100
Which mechanism of injury across the elbow leads to this type of dislocation?
1- Axial compression, forearm supination, valgus stress
2- Axial compression, forearm supination, varus stress
3- Axial compression, forearm pronation, valgus stress
4- Axial distraction, forearm pronation, varus stress

PREFERRED RESPONSE: 1- Axial compression, forearm supination, valgus stress

Question 52 of 100
The patient is in the emergency department, and the orthopaedic surgeon is performing a
closed reduction under conscious sedation. Following the reduction, what is the most stable
position in which to splint the arm to prevent further subluxation?
1- Elbow flexion, supination

43
AAOS 2016 Sports medicine

2- Elbow flexion, pronation


3- Elbow extension, supination
4- Elbow extension, pronation

PREFERRED RESPONSE: 2- Elbow flexion, pronation

Question 53 of 100
The elbow is successfully reduced in the emergency department and placed in a splint.
What is the most appropriate next step in this evaluation?
1- MRI
2- CT scan
3- Electromyography
4- Radiographs

PREFERRED RESPONSE: 4- Radiographs

Question 54 of 100
No fractures were identified and the patient was treated nonsurgically in a range-of-motion
brace. Two months later, he continued to experience elbow pain and was unable to return to
sports. He regained motion and strength with physical therapy, there was no gross instability
with varus or valgus testing, and he had a negative moving-valgus stress test. The orthopaedic
surgeon performed an examination under anesthesia in the operating room (Video 54). Which
anatomic structure is injured?
1- Medial ulnar collateral ligament (UCL) (anterior bundle)
2- Medial UCL (posterior bundle)
3- Lateral UCL
4- Annular ligament

PREFERRED RESPONSE: 3- Lateral UCL

44
AAOS 2016 Sports medicine

DISCUSSION
Ninety percent of elbow dislocations occur in a posterolateral direction. ODriscoll and
associates described the mechanism of injury in posterolateral elbow dislocations in 1992,
reporting that they occur most typically after a fall onto an outstretched arm. As the arm hits
the ground it causes axial compression, forearm supination, and valgus load across the elbow.
The triceps fires, pulling the olecranon posterior; the forearm supinates while simultaneous
shoulder internal rotators fire; and the elbow falls into valgus. These 3 mechanisms cause the
elbow to subluxate and dislocate posterolaterally. The elbow is most stable following
posterolateral dislocation in a flexed and pronated position. The elbow is least stable in
extension and supination. Simple dislocation often can be treated nonsurgically, while fracture
dislocation will usually necessitate surgical intervention. The video shows the elbow pivot-
shift test, which evaluates for posterolateral rotatory instability. A positive test finding elicits
apprehension and, in this case, radial head subluxation and confirms an insufficient lateral
UCL.

Question 55 of 100
A 19-year-old collegiate lacrosse player has bilateral lower extremity pain during training
runs and practice sessions. She says her pain is only associated with activity, always begins 4
to 5 minutes into an activity, and resolves within 10 minutes after activity cessation. She
reports a feeling of numbness and tingling in her first- and second-toe web space with
continued activity. She has no pain while at rest and has tried nonsteroidal anti-inflammatory
drugs, ice baths, and rehabilitation techniques with her athletic trainers without experiencing
noticeable changes. Which test has the highest specificity and sensitivity for the pathology
suspected?
1- MRI of the affected tibia
2- Palpation of the affected leg compartments during activity
3- Compartment pressure monitoring 1 and 5 minutes into the activity
4- Compartment pressure monitoring continuously during the activity

PREFERRED RESPONSE: 4- Compartment pressure monitoring continuously during the


activity

DISCUSSION
Chronic exertional compartment syndrome (CECS) is an uncommon cause of pain that is
most often encountered in athletes or military populations. The diagnosis is suggested by pain
after the start of the activity with resolution of symptoms when the activity stops. Traditionally,
the diagnosis was established by measuring intracompartment pressures at 1 and 5 minutes of

45
AAOS 2016 Sports medicine

activity. Roscoe and associates have shown that using continuous pressure monitoring during
the activity until pain forces subjects to stop provides higher sensitivity and specificity than
the traditional diagnostic technique. Palpating the leg is not as sensitive or specific as direct
pressure monitoring. Although certain changes can be seen on MRI in CECS, this is not the
diagnostic study of choice.

Figure 56

Question 56 of 100
Figure 56 is the MR image of a 20-year-old Division I baseball pitcher who has a 1-month
history of medial elbow pain in his throwing arm. He also notes a decrease in both control and

46
AAOS 2016 Sports medicine

pitching velocity. An examination reveals tenderness at the medial epicondyle that is


exacerbated with valgus elbow stress. The strongest indication for ulnar collateral ligament
(UCL) reconstruction is
1- progressive ulnar neuropathy.
2- a decision to enter the Major League Baseball (MLB) draft.
3- pain with resisted wrist flexion.
4- failure to improve after prolonged nonsurgical treatment.

PREFERRED RESPONSE: 4- failure to improve after prolonged nonsurgical treatment.

DISCUSSION
All responses represent findings that may be associated with chronic UCL insufficiency.
Responses 1 and 3 reflect injury to the UCL itself. In most patients, particularly young patients,
UCL reconstruction should not be considered until an appropriate trial of nonsurgical measures
has failed. This trial should include, at a minimum, 6 weeks of throwing abstinence followed
by rehabilitation to address pitching mechanics and shoulder motion deficits and core
strengthening. Although the decision to enter the MLB draft may influence surgical decision
making, a pitcher with a 1-month history of elbow symptoms should attempt nonsurgical
therapy before making a surgical decision that is not based on clinical data.

Question 57 of 100
A coach of 3 football teamsthe B team, junior varsity team, and varsity teamwants to
study the average times in the 40-yard dash for his players. Which test would help him
determine if the mean 40-yard dash times for the athletes on 1 team are different from those
on the other teams?
1- Independent 2-sample t test
2- Analysis of variance (ANOVA)
3- Chi-square test
4- Fishers exact test

PREFERRED RESPONSE: 2- Analysis of variance (ANOVA)

DISCUSSION

47
AAOS 2016 Sports medicine

Data collected in research studies fall into 1 of 2 categoriescontinuous or discrete.


Continuous data can be displayed on a curve. Examples include height, weight, and time
recorded in a 40-yard dash. Discrete data represent data that fall into specific categories such
as gender or the presence or absence of a risk factor.
ANOVA is used to determine statistical significance in mean values of continuous data
when there are more than 2 independent samples. The 2-sample t test compares mean values
of continuous data between 2 independent groups. The Chi-square test and Fisher's exact tests
are tests used to analyze discrete data.

RESPONSES FOR QUESTIONS 58 THROUGH 61


1- Paresthesias in the fourth and fifth digits
2- Numbness on the lateral side of the forearm
3- Heterotopic ossification
4- Posterolateral rotatory instability of the elbow
5- Medial antebrachial cutaneous neuroma
For each surgical case described below, match the most likely related complication listed
above.

Question 58 of 100
2-incision distal biceps repair
1- Paresthesias in the fourth and fifth digits
2- Numbness on the lateral side of the forearm
3- Heterotopic ossification
4- Posterolateral rotatory instability of the elbow
5- Medial antebrachial cutaneous neuroma

PREFERRED RESPONSE: 3- Heterotopic ossification

Question 59 of 100
Single-incision distal biceps repair
1- Paresthesias in the fourth and fifth digits
2- Numbness on the lateral side of the forearm

48
AAOS 2016 Sports medicine

3- Heterotopic ossification
4- Posterolateral rotatory instability of the elbow
5- Medial antebrachial cutaneous neuroma

PREFERRED RESPONSE: 2- Numbness on the lateral side of the forearm

Question 60 of 100
Arthroscopic lateral epicondylitis debridement
1- Paresthesias in the fourth and fifth digits
2- Numbness on the lateral side of the forearm
3- Heterotopic ossification
4- Posterolateral rotatory instability of the elbow
5- Medial antebrachial cutaneous neuroma

PREFERRED RESPONSE: 4- Posterolateral rotatory instability of the elbow

Question 61 of 100
Ulnar collateral ligament (UCL) reconstruction using a modified Jobe technique
1- Paresthesias in the fourth and fifth digits
2- Numbness on the lateral side of the forearm
3- Heterotopic ossification
4- Posterolateral rotatory instability of the elbow
5- Medial antebrachial cutaneous neuroma

PREFERRED RESPONSE: 1- Paresthesias in the fourth and fifth digits

DISCUSSION
Certain complications are more strongly associated with the approach and surgical
procedure for elbow pathology. With a 2-incision distal biceps repair, heterotopic ossification

49
AAOS 2016 Sports medicine

with a radial-ulnar synostosis is a concern. This complication can be minimized through


irrigation of bone debris and care to avoid dissection between the radius and ulna. With a
single-incision distal biceps repair, the lateral antebrachial cutaneous nerve is retracted during
the procedure. Numbness on the lateral side of the forearm is common, although often
temporary. During arthroscopic debridement for lateral epicondylitis, injury to the radial UCL
can occur, leading to posterolateral rotatory instability of the elbow. The modified Jobe
technique for UCL reconstruction typically involves an ulnar nerve transposition during the
procedure. Numbness and tingling in the fourth and fifth digits are concerns when this
procedure is performed.

Question 62 of 100
A 17-year-old high school football player sustains a neck injury in a game. During the
initial on-field assessment, the team physician removes his helmet, and the athlete is log-rolled
to the supine position while the physician manually stabilizes his cervical spine. An
examination demonstrates tenderness to palpation over the cervical spine and neurologic
deficits in bilateral upper and lower extremities. Shoulder pads prohibit proper placement of a
hard cervical collar, and the athlete is immobilized on a spine board and transported to the
emergency department via ambulance. Comprehensive evaluation in the emergency
department reveals a bilateral facet dislocation of C5 on C6. The on-field intervention most
likely to cause a neurologic injury is
1- failure to place a hard cervical collar.
2- helmet removal prior to examination.
3- transfer to a spine board prior to transport.
4- log-rolling the athlete to the supine position.

PREFERRED RESPONSE: 2- helmet removal prior to examination.

DISCUSSION
Complete immobilization of the cervical spine is critical for athletes with a suspected
cervical spine or spinal cord injury. The spinal cord in the subaxial spine is especially sensitive
to motion, and removal of protective gear such as the helmet and shoulder pads presents an
unacceptable risk for progressive neurologic injury in the setting of a potentially unstable
cervical spine injury. Removal of the face mask alone is typically performed to improve access
to an athlete's airway. Protective equipment often prevents proper placement of a hard cervical
collar, and the spine board offers a variety of options for safe cervical spine immobilization of
helmeted athletes without a hard cervical collar. The log-roll and lift-and-slide techniques

50
AAOS 2016 Sports medicine

allow for the safe transfer of an athlete to a spine board while maintaining appropriate manual
stabilization of the cervical spine.

Question 63 of 100
Which group experiences the highest rate of anterior cruciate ligament (ACL) tears?
1- Female athletes with valgus knee alignment and small femoral notch width
2- Female athletes with valgus knee alignment and large ACL width
3- Male athletes with valgus knee alignment and small ACL width
4- Male athletes with varus knee alignment and small femoral notch width

PREFERRED RESPONSE: 1- Female athletes with valgus knee alignment and small
femoral notch width

DISCUSSION
ACL tears are several times more common among women than men. Women who land
from jumps in increased valgus and external rotation are at particularly increased risk for ACL
tears. Women have smaller notch widths and a smaller ACL cross-sectional area than men,
but these factors have not been proven to increase risk for ACL tears.

51
AAOS 2016 Sports medicine

Figure 64

CLINICAL SITUATION FOR QUESTIONS 64 THROUGH 67


Figure 64 is the radiograph of a 21-year-old college lacrosse player who has a 2-year
history of progressive left groin pain that is exacerbated by activity. Pain is preventing him
from participating with his team. Examination reveals a fit man without tenderness to palpation
around the hip. No clicking or popping occurs with hip range of motion. Strength of all muscles
about the hip is normal, but there is some mild pain with resisted hip flexion and hip adduction.
While lying supine, progressive hip flexion with internal rotation and adduction reproduces
his groin pain.

Question 64 of 100
What is the most likely cause of this patients pain?
1- Femoroacetabular impingement (FAI)
2- Osteoarthritis of the sacroiliac joint
3- Intra-articular loose body
4- Trochanteric bursitis

PREFERRED RESPONSE: 1- Femoroacetabular impingement (FAI)

Question 65 of 100
The patient participates in physical therapy for 8 weeks with his teams trainer but notes
little improvement. What is the most appropriate next diagnostic step to determine the cause
of his pain?
1- Diagnostic arthroscopy of the hip
2- Hip bone scan
3- Hip MRI arthrogram
4- Hip ultrasound

PREFERRED RESPONSE: 3- Hip MRI arthrogram

Question 66 of 100

52
AAOS 2016 Sports medicine

Further workup confirms an anterosuperior tear of the acetabular labrum and prominence
of the acetabulum. What is the most likely location of a chondral injury associated with these
findings?
1- Posterosuperior acetabulum
2- Posteroinferior acetabulum
3- Femoral head above the fovea
4- Femoral head below the fovea

PREFERRED RESPONSE: 2- Posteroinferior acetabulum

Question 67 of 100
The patient experiences little improvement with activity modification and more physical
therapy. An intra-articular corticosteroid injection provides excellent relief, but relief only lasts
for 1 month. The player requests further treatment for his hip and is counseled regarding
surgical intervention. Hip arthroscopy is performed. Intrasurgically, a capsulolabral separation
is observed with an underlying pincer lesion. No articular cartilage injury is seen. Which
treatment is most appropriate considering these findings?
1- Debridement of the labral tear plus bony resection of the pincer lesion
2- Debridement of the labral tear and no bony resection of the pincer lesion
3- Femoral neck osteoplasty plus labral repair using suture anchor
4- Resection of the bony pincer lesion plus labral repair using suture anchor

PREFERRED RESPONSE: 4- Resection of the bony pincer lesion plus labral repair using
suture anchor

DISCUSSION

53
AAOS 2016 Sports medicine

Video 67 for reference

This clinical scenario describes a patient with FAI attributable to pincer (acetabular)
deformity. This form of FAI, which involves prominence of the anterosuperior acetabular lip,
may be more common among women. Decreased range of motion and pain occur secondary
to the abutment of the femoral head against the acetabular labrum and rim. Hip flexion,
combined with adduction and internal rotation, recreates this contact and causes pain, but
CAM or pincer etiology remains unknown.
The differential diagnosis of hip pain in a young athlete includes femoral neck stress
reaction/fracture, sacroiliac arthritis, intra-articular loose body, trochanteric bursitis, osteitis
pubis, and hernia. No information presented in this scenario suggests any of these causes.
Diagnosis of FAI is best performed via MR imaging, with an arthrogram increasing the
sensitivity and specificity for labral pathology. An ultrasound may be useful in the diagnosis
of dysplasia or for dynamic assessment of a snapping hip, but ultrasound is not commonly
used to diagnose labral pathology.
Although concomitant chondral lesions of the femoral head are uncommon, the forced
leverage of the anterosuperior femoral neck upon the anterior acetabulum may result a contra-
coup chondral injury on the posteroinferior acetabulum. This is the most common location of
chondral lesions in this scenario. Without bony resection to prevent further impingement, this
patient will continue to experience symptoms. Because there is no evidence of femoral neck
prominence (CAM lesion), there is no indication for osteoplasty of the femoral neck; resection
of the pincer lesion is necessary. This will often require take-down of the labrum in this
location. If possible, iatrogenic or traumatic labral tears should subsequently be repaired after
pincer debridement because the labrum has important functions for hip stability and
maintenance of the suction seal of the joint.

Question 68 of 100

54
AAOS 2016 Sports medicine

A 26-year-old weight lifter has had increasing pain in his left shoulder for 4 months. His
symptoms do not improve with nonsurgical treatment that included activity modification, anti-
inflammatory medication, and corticosteroid injections. He undergoes arthroscopic distal
clavicle excision with resection of the distal 2.5 cm of clavicle. Three months after surgery, he
reports persistent pain and popping in his shoulder. An examination demonstrates anterior and
posterior instability of the distal clavicle without gross deformity. Radiographs are
unremarkable. What is the most likely cause of distal clavicle instability after surgery?
1- Release of the coracoacromial ligament
2- Use of an arthroscopic rather than open technique
3- Overresection of the distal clavicle
4- Treatment with corticosteroid injections before surgery

PREFERRED RESPONSE: 3- Overresection of the distal clavicle

DISCUSSION
Overresection of the distal clavicle can result in disruption of the acromioclavicular
ligamentous complex, which inserts at an average of 22.9 mm from the distal clavicle. A
comparison of arthroscopic and open distal clavicle excision demonstrated less pain in the
arthroscopic group, with no difference in patient satisfaction or shoulder function between
groups. Injuries to the conoid and trapezoid ligaments occur with high-grade
acromioclavicular separations, resulting in superior migration of the distal clavicle relative to
the acromion. Release of the coracoacromial ligament typically is not performed during distal
clavicle excision.

55
AAOS 2016 Sports medicine

Figure 69a Figure 69b

56
AAOS 2016 Sports medicine

Figure 69c Figure 69d

Figure 69e

Question 69 of 100
The radiographic finding indicated by the arrow in Figure 69a is most commonly
associated with which arthroscopic image?
1- Figure 69b
2- Figure 69c
3- Figure 69d
4- Figure 69e

57
AAOS 2016 Sports medicine

PREFERRED RESPONSE: 2- Figure 69c

DISCUSSION
The Segond fracture, first described in cadaveric dissections by Paul Segond in 1879, is a
lateral capsular avulsion fracture occurring just distal to the lateral tibial plateau. It is
associated with tears of the anterior cruciate ligament in more than 75% of cases.
Radiographically, this lateral capsular sign is easily identified on a standard anteroposterior
view of the affected knee, but it also may be seen on a CT scan or MRI.
In 2013, a distinct, lateral capsular ligament known as the anterolateral ligament (ALL)
was described. The ligament originates on the lateral femoral epicondyle and inserts on the
anterolateral tibia. Claes and associates showed that the ALL inserts in the region on the
proximal tibia from which Segond fractures consistently avulse, suggesting that a Segond
fracture is actually a bony avulsion of the ALL.

Figure 70

Question 70 of 100
Figure 70 is the radiograph of a 14-year-old girl with increasing posterior ankle pain,
especially during pointe technique exercises. Nonsurgical measures such as modification,
stretching, and injection have been unsuccessful. Which nerve is most vulnerable to injury
during endoscopic excision of this lesion?

58
AAOS 2016 Sports medicine

1- Sural
2- Deep peroneal
3- Medial plantar
4- Posterior tibial

PREFERRED RESPONSE: 1- Sural

DISCUSSION
The pathology is that of posterior ankle impingement, which is secondary to a symptomatic
os trigonum. Endoscopic excision necessitates posteromedial and posterolateral ankle portals.
Although plantar numbness has been described as a relatively common postsurgical
complication, the neurovascular structure most commonly injured is the sural nerve.

Figure 71a Figure 71b

Question 71 of 100
Figures 71a and 71b/ are the MR images of a 65-year-old man who dislocated his shoulder.
What is his most likely chief symptom?
1- Numbness in the anterior aspect of his shoulder

59
AAOS 2016 Sports medicine

2- Recurrent instability
3- Difficulty raising his arm
4- Biceps muscle deformity

PREFERRED RESPONSE: 3- Difficulty raising his arm

DISCUSSION
This patient has a massive rotator cuff tear resulting in disruption of the transverse force
couple between the subscapularis anteriorly and the infraspinatus and teres minor posteriorly.
These muscles provide dynamic shoulder stability throughout active elevation. Loss of the
force couple produces a pathologic increase in translation of the humeral head and decreased
active abduction and external rotation, which results in difficulty raising an arm. The most
common neurologic deficit after shoulder dislocation is isolated injury to the axillary nerve
that supplies sensation to the lateral aspect of the shoulder, not the anterior aspect. Recurrent
instability is uncommon unless there is a labral tear or massive subscapularis tear. The biceps
muscle is not viewed in the MR images, and a complete proximal biceps tendon rupture would
be uncommon in the setting of an anterior shoulder dislocation.

Figure 72a Figure 72b

60
AAOS 2016 Sports medicine

Figure 72c Figure 72d

Figure 72a

CLINICAL SITUATION FOR QUESTIONS 72 THROUGH 75


Figures 72a through 72e are the MR images of a 12-year-old boy with left lateral-sided
knee pain following a football injury. He has a several-year history of recurrent knee pain that
improves with rest. An examination reveals a moderate effusion. Range of motion is 0 degrees

61
AAOS 2016 Sports medicine

to 90 degrees and is limited by pain in deep flexion. He has tenderness to palpation along the
lateral joint line, and no instability is noted.

Question 72 of 100
What is the underlying cause of the pathology noted in the figures?
1- Genetic mutation
2- Recurrent trauma
3- Shallow intercondylar notch
4- Congenital abnormality

PREFERRED RESPONSE: 4- Congenital abnormality

Question 73 of 100
Based on the pathology noted, which finding may be found on plain knee radiographs?
1- Shallow trochlear groove
2- Squaring of the lateral femoral condyle
3- Deepening of the sulcus terminalis
4- Medial joint space narrowing

PREFERRED RESPONSE: 2- Squaring of the lateral femoral condyle

Question 74 of 100
What other finding may be noted in patients with this diagnosis?
1- Symmetric knee pathology
2- Excessive joint laxity
3- Recurrent patella instability
4- Extra-articular manifestations

PREFERRED RESPONSE: 1- Symmetric knee pathology

62
AAOS 2016 Sports medicine

Question 75 of 100
Treatment should include
1- anterior cruciate ligament reconstruction with lateral meniscus repair.
2- partial lateral meniscectomy with saucerization.
3- lateral meniscus transplant.
4- protected weight bearing with referral for genetic testing.

PREFERRED RESPONSE: 2- partial lateral meniscectomy with saucerization.

DISCUSSION
The MR images show a tear through a discoid lateral meniscus. A discoid meniscus is
caused by a failure of apoptosis during development in utero and is considered a congenital
abnormality. Discoid menisci are prone to tearing and can be caused by minimal trauma. There
is no known genetic cause for this condition. Radiographic findings that may be present in the
setting of a discoid meniscus include lateral joint space widening, squaring of the lateral
femoral condyle, and cupping of the lateral tibial plateau. Contralateral discoid menisci are
noted in 20% of patients. There are no other known associated conditions. Treatment of a
symptomatic discoid meniscus should include partial meniscectomy and saucerization or
repair.

63
AAOS 2016 Sports medicine

Figure 76

Question 76 of 100
Figure 76 is the MR image of a 16-year-old high school football player who sustained a
traumatic dominant shoulder dislocation during a game. On-field reduction was unsuccessful.
The shoulder is reduced in the emergency department, and the player and his family follow-
up in clinic. Which factor is most associated with failure of surgical treatment in this scenario?
1- Dominant shoulder
2- Age
3- Size of lesion
4- Periosteal stripping

PREFERRED RESPONSE: 2- Age

DISCUSSION
The MR image reveals a Bankart lesion. Arthroscopic Bankart repair failure likelihood is
increased by numerous factors. Age, number of recurrences, and bony defects are most
associated with failure of arthroscopic repair. Shoulder dominance, amount of periosteal

64
AAOS 2016 Sports medicine

stripping, and difficulty of reduction do not correlate with increased recurrence risk following
surgery.

CLINICAL SITUATION FOR QUESTIONS 77 THROUGH 79


A 17-year-old volleyball player has a 3-month history of gradually worsening right
shoulder pain. She describes a vague sensation of her shoulder popping out of place and
weakness associated with overhead activities. She has intermittent generalized paresthesias in
her right upper extremity, and she has discontinued participation in sports as a result of her
symptoms. Glenohumeral range of motion is symmetric bilaterally. Empty can test findings
are negative with full strength of the supraspinatus. An active compression test is negative,
and sulcus sign findings are positive. An anterior apprehension test produces pain that is
unrelieved with a relocation test. A cervical spine examination is unremarkable.

Question 77 of 100
Which examination finding most likely is present?
1- Lateral scapular winging
2- Positive Mayo shear test
3- Grade 3 instability on an anterior load-and-shift test
4- Scapulothoracic dyskinesia

PREFERRED RESPONSE: 4- Scapulothoracic dyskinesia

Question 78 of 100
Treatment with muscle strengthening exercises for the rotator cuff and scapular stabilizers
most likely will result in
1- diminished pain and improved stability.
2- glenohumeral joint dislocation.
3- progression of neurologic symptoms.
4- normal muscle activation and motion patterns.

PREFERRED RESPONSE: 1- diminished pain and improved stability.

65
AAOS 2016 Sports medicine

Question 79 of 100
The patient does not improve with 1 year of rehabilitation exercises. MR arthrography
reveals a normal glenoid labrum and rotator cuff. Surgical treatment should consist of
1- rotator interval closure.
2- thermal capsulorrhaphy.
3- arthroscopic capsular plication.
4- suprascapular nerve decompression.

PREFERRED RESPONSE: 3- arthroscopic capsular plication.

DISCUSSION
This patient has multidirectional instability (MDI). Symptoms are typically of insidious
onset with nonspecific sports-related pain during the second or third decade of life. The
etiology of MDI involves a patulous inferior capsular complex, but, in isolation, this lesion
may not produce symptoms. Patients with MDI have abnormal patterns of rotator cuff muscle
activity that is not restored with nonsurgical treatment. Symptomatic patients with MDI also
demonstrate increased rates of abnormal scapular kinematics. The prevalence of MDI is higher
among overhead athletes. The sulcus sign is an examination finding that produces a visible
dimple inferior to the lateral border of the acromion with application of inferior traction on the
arm. Generalized hyperlaxity or a connective tissue disorder may be present. Physical therapy
for strengthening of the rotator cuff and scapular stabilizers remains the recommended initial
treatment. Rehabilitation should continue for at least 6 months (and possibly much longer).
Motivated patients frequently report diminished pain and improved stability with
strengthening exercises. If nonsurgical measures fail to provide adequate relief, arthroscopic
capsular plication is a viable treatment option, with high rates of return to play among properly
selected patients. Thermal capsulorrhaphy has a high failure rate and poses potential for
serious complications, including chondrolysis and thermal injury to the axillary nerve. Closure
of the rotator interval has not been definitively shown to enhance stability or improve
outcomes for patients with MDI.

66
AAOS 2016 Sports medicine

Figure 80a Figure 80b

Question 80 of 100
Figures 80a and 80b are the radiographs of a 21-year-old football player who underwent
anterior cruciate ligament (ACL) reconstruction with patellar tendon autograft 1 year ago. He
reports mild stiffness in his knee. Upon examination, he has a negative Lachman test result,
trace effusion, and range of motion from 0 to 85 degrees of knee flexion. Which factor is most
contributory to his examination findings?
1- Incorrect graft choice
2- Improper tunnel position
3- Tibial graft-tunnel mismatch

67
AAOS 2016 Sports medicine

4- Poor femoral fixation

PREFERRED RESPONSE: 2- Improper tunnel position

DISCUSSION
Technical failure is the most common reason for ACL reconstruction failure. Tunnel
position is the most frequent cause of technical failure. Malpositioning of the tunnel affects
the length of the graft, causing either decreased range of motion or increased graft laxity. This
patient has anterior and vertical placement of his femoral tunnel, which has been shown to
cause stiffness in knee flexion. Although graft choice is an important factor when planning
ACL reconstruction, overall outcomes with autograft tissues are fairly similar. Fixation of the
graft at the femoral or tibial end is not as important as tunnel position. Fixing the graft in
flexion can cause extension loss when isometry is not achieved, but this condition is not
touched upon in this scenario.

Figure 81a Figure 81b

68
AAOS 2016 Sports medicine

Figure 81c Figure 81d

CLINICAL SITUATION FOR QUESTIONS 81 THROUGH 84


Figures 81a through 81d are the MR images of a 25-year-old man with left knee pain after
a motorcycle collision. He has palpable pedal pulses with an ankle-brachial index of 0.95.
Neurologic examination findings of the injured extremity are normal.

Question 81 of 100
Based on Figures 81a through 81d, which structures are injured?
1- Anterior cruciate ligament (ACL) and posterior cruciate ligament (PCL)
2- ACL, PCL, and medial collateral ligament
3- ACL and lateral collateral ligament
4- ACL, PCL, and posterolateral corner (PLC)

PREFERRED RESPONSE: 4- ACL, PCL, and posterolateral corner (PLC)

Question 82 of 100
Surgical treatment within 3 weeks of injury offers which advantage over delayed surgical
treatment?
1- Increased rate of return to work

69
AAOS 2016 Sports medicine

2- Higher subjective outcome scores


3- Less residual anterior knee instability
4- Reduced need to undergo a second surgery

PREFERRED RESPONSE: 2- Higher subjective outcome scores

Question 83 of 100
Which factor is most closely associated with increased risk for surgical complications in
this scenario?
1- Age
2- Morbid obesity
3- Injury mechanism
4- Time to surgery

PREFERRED RESPONSE: 2- Morbid obesity

Question 84 of 100
Surgical arthroscopy performed 1 week after injury presents increased risk for
1- deep venous thrombosis.
2- popliteal artery dissection.
3- complex regional pain syndrome.
4- compartment syndrome.

PREFERRED RESPONSE: 4- compartment syndrome.

DISCUSSION
The MR images show injuries to the ACL, PCL, and PLC consistent with a knee
dislocation. The optimal timing of surgery after multiligament knee injury remains unclear.
Two systematic reviews demonstrated superior clinical outcome scores after early treatment,
including higher mean Lysholm scores and a higher percentage of good/excellent International

70
AAOS 2016 Sports medicine

Knee Documentation Committee scores. Early treatment was associated with increased
residual anterior knee instability but no difference in posterior instability, varus laxity, or
valgus laxity. Although numbers were limited, the average range of motion and rate of
extension loss of at least 5 degrees was similar between groups. More patients in the early-
treatment group demonstrated a higher rate of flexion loss of 10 or more degrees and an
increased need to undergo a second procedure to address arthrofibrosis, including
manipulation under anesthesia and arthrolysis. Return to work did not significantly differ
between groups, but return to sports was lower in the early-treatment group. Evidence
demonstrates a higher rate of low-energy mechanisms resulting in multiligament knee injury
and an increased odds ratio for complications among obese (= 30 degrees kg/m2 patients,
including wound complications and neurovascular injury. The complication rate increased
9.2% for every 1-point increase in body mass index. There is no association between
complication rate and age, injury mechanism, or timing of surgery. Orthopaedic surgeons
performing arthroscopy during the early postinjury period must be mindful of the extensive
soft-tissue damage present in these patients, including potential capsular defects. Use of high-
pressure irrigation can lead to substantial fluid extravasation into the thigh or lower leg
compartments, placing patients at increased risk for compartment syndrome. In addition to
avoiding high-pressure irrigation, some orthopaedic surgeons have advocated the creation of
generous capsular incisions during portal establishment to allow for ready egress of irrigation
fluid from the portal sites rather than into soft tissues.

RESPONSES FOR QUESTIONS 85 THROUGH 88


1- Excessive medial placement of coracoid autograft
2- Excessive lateral placement of coracoid autograft
3- Excessive inferior dissection during the procedure
4- Excessive retraction and dissection of the medial portion of the conjoint tendon
A 20-year-old right-hand-dominant football player sustained a traumatic shoulder
dislocation during a tackle. He has had multiple recurrent dislocations, and radiographs reveal
anterior glenoid bone loss. He underwent a Latarjet procedure. Match the most likely
complication described below with the surgical error listed above.

Question 85 of 100
The patient sustains a recurrent dislocation.
1- Excessive medial placement of coracoid autograft
2- Excessive lateral placement of coracoid autograft
3- Excessive inferior dissection during the procedure
4- Excessive retraction and dissection of the medial portion of the conjoint tendon

71
AAOS 2016 Sports medicine

PREFERRED RESPONSE: 1- Excessive medial placement of coracoid autograft

Question 86 of 100
The patient is unable to actively elevate his arm and has atony of the deltoid.
1- Excessive medial placement of coracoid autograft
2- Excessive lateral placement of coracoid autograft
3- Excessive inferior dissection during the procedure
4- Excessive retraction and dissection of the medial portion of the conjoint tendon

PREFERRED RESPONSE: 3- Excessive inferior dissection during the procedure

Question 87 of 100
Two years after undergoing the procedure, the patient develops shoulder crepitus, and
radiographs reveal arthritis.
1- Excessive medial placement of coracoid autograft
2- Excessive lateral placement of coracoid autograft
3- Excessive inferior dissection during the procedure
4- Excessive retraction and dissection of the medial portion of the conjoint tendon

PREFERRED RESPONSE: 2- Excessive lateral placement of coracoid autograft

Question 88 of 100
The patient has weakness with elbow flexion and has numbness down the anterior lateral
aspect of the forearm.
1- Excessive medial placement of coracoid autograft
2- Excessive lateral placement of coracoid autograft
3- Excessive inferior dissection during the procedure
4- Excessive retraction and dissection of the medial portion of the conjoint tendon

72
AAOS 2016 Sports medicine

PREFERRED RESPONSE: 4- Excessive retraction and dissection of the medial portion


of the conjoint tendon

DISCUSSION
The Latarjet procedure was initially described in 1959 as a modification of the Bristow
procedure. It has been used as a primary procedure to address instability, but is used more
commonly for patients with instability and glenoid bone loss. In 2000, Burkhart and associates
reported a 67% failure rate of the Bankart procedure in patients with an inverted pear-shaped
glenoid (glenoid bone loss) or an engaging Hill-Sachs lesion, with a suggestion that a bone
graft procedure would be optimal in this population. Complications following the Latarjet
procedure have been reported as high as 25%, with the majority attributable to nerve injury,
recurrent instability, and arthritis. Many of these complications are likely secondary to surgical
technique. A coracoid graft that is placed too laterally or with prominent screws will overhang
the glenoid and lead to early degenerative glenohumeral arthritis. A coracoid graft placed too
medially can lead to recurrent instability secondary to an ineffective subscapularis sling and
bone block. A coracoid graft placed inferiorly indicates dissection close to the axillary nerve,
which can place tension on the axillary nerve or cause injury from direct trauma. After
harvesting the coracoid graft, the surgeon must find the musculocutaneous nerve as it enters
the conjoint tendon on the medial surface about 5 cm distal to the coracoid. Excessive
dissection or retraction can lead to musculocutaneous nerve palsy.

Question 89 of 100
A 27-year-old man has recurrent right shoulder instability. He first dislocated his shoulder
in college while playing rugby and was treated nonsurgically. Since then, he has sustained
nearly 1 dozen dislocations and says that his shoulder always feels loose. The shoulder
recently dislocated in his sleep and while he was putting on clothes. Which factor is a
contraindication to an arthroscopic soft-tissue repair?
1- A 270-degree labral tear
2- His intention to continue contact sport activities
3- Anterior bony loss measuring 30% of inferior glenoid width
4- An inferior glenohumeral ligament avulsion (HAGL) lesion

PREFERRED RESPONSE: 3- Anterior bony loss measuring 30% of inferior glenoid


width

73
AAOS 2016 Sports medicine

DISCUSSION
There is much debate in the literature regarding optimal techniques for treatment of
shoulder instability. Although some studies suggest that open stabilization may result in lower
recurrence rates in contact athletes, this approach is now under scrutiny. Extensive labral
involvement (posterior labral involvement in this scenario) is likely more accessible via
arthroscopic methods. Although HAGL lesions may be more easily accessible via an open
approach (particularly for inexperienced arthroscopists), numerous authors describe successful
repair via arthroscopic techniques. Among these responses, the strongest indication for an open
approach, including possible bony transfer, is high-grade glenoid bone loss. Although the
critical amount of bone loss is a topic of debate, most surgeons and authors suggest a cutoff of
20% to 25%.

Figure 90

CLINICAL SITUATION FOR QUESTIONS 90 THROUGH 92


Figure 90 is the radiograph of a 14-year-old pitcher who plays in a year-round baseball
program and has vague pain in his dominant shoulder. The pain occurs with throwing, and it
has been worsening for 2 months. Pain typically occurs during the late cocking phase of
throwing. He has no tenderness of the rotator cuff and 5/5 rotator cuff strength. His arc of
motion is symmetric between his dominant and nondominant arms. The sulcus sign is negative.

74
AAOS 2016 Sports medicine

Question 90 of 100
What is the most likely diagnosis?
1- Proximal humeral epiphysiolysis
2- Internal impingement with internal rotation deficit
3- Rotator cuff tendinitis
4- Biceps tendinitis

PREFERRED RESPONSE: 1- Proximal humeral epiphysiolysis

Question 91 of 100
Initial treatment should consist of
1- arthroscopic labral repair
2- rest and physical therapy
3- immobilization in external rotation
4- closed reduction and percutaneous pinning.

PREFERRED RESPONSE: 2- rest and physical therapy

Question 92 of 100
What is the most likely contributory factor to this patient's problem?
1- Throwing curveballs
2- Year-round throwing
3- Poor calcium intake
4- Shoulder laxity

PREFERRED RESPONSE: 2- Year-round throwing

DISCUSSION

75
AAOS 2016 Sports medicine

Proximal humeral epiphysiolysis (little leaguers shoulder) is an overuse condition of the


proximal humeral physis. Patients report diffuse pain that is worse with throwing. Little
leaguers shoulder is caused by rotational stress placed on the proximal humeral epiphysis
during overhead throwing. The growth plate is weakest to torsion stress and is most susceptible
to injury during periods of rapid growth commonly seen during puberty. Most chronic shoulder
injuries occur in throwing athletes between 13 and 16 years of age. Factors that contribute to
the condition include excessive throwing, improper throwing mechanics, and muscle-tendon
imbalance. Radiographic findings typically are normal but may indicate subtle widening of
the proximal humeral physis, and, in more severe cases, metaphyseal demineralization or
fragmentation. Surgical fixation is not required for healing. An initial 3-month period of rest
and activity modification will typically result in resolution of symptoms. Nonsteroidal anti-
inflammatory drugs may be used as needed. After the rest period, a gradual return to throwing
is implemented until the patients condition returns to baseline. This protocol has a long-term
success rate exceeding 90%. Pitching coaches should evaluate throwing mechanics and
maintain pitch counts. The most common cause of this condition is overuse, as is seen in
pitchers who throw all year. Internal rotation deficit and internal impingement is typically a
finding in older athletes without open physes. This patient had no evidence of rotation deficit
upon examination, making this diagnosis unlikely.

Question 93 of 100
A 17-year-old African American high school football player is in the afternoon session of
an August 2-a-day practice. He tells his trainer he is experiencing weakness, dizziness, and
nausea. The ambient temperature is 31C with a relative humidity of 70%. An examination by
the team trainer reveals a body temperature of 39C and headache, chills, confusion, and
disorientation. What is the most likely diagnosis?
1- Sickle-cell crisis
2- Heat exhaustion
3- Heatstroke
4- Dehydration

PREFERRED RESPONSE: 3- Heatstroke

DISCUSSION
Heat exhaustion and heatstroke are both forms of heat illness during which the body is
unable to self-regulate internal temperature. The hallmarks of heatstroke are altered mental
status and/or core temperature higher than 40C. Heat exhaustion may be marked by nausea,
vomiting, headache, dizziness, chills, and excessive sweating, but there are no mental status

76
AAOS 2016 Sports medicine

changes. In heatstroke, sweating can often slow or cease as dysregulation worsens. Simple
dehydration would not result in mental status changes or elevated core temperature. Sickle-
cell crisis is marked by extreme pain, with location depending on the site of crisis. Four main
patterns are common: bone, chest, abdominal, or joint crises. Sickle-cell crisis can be
precipitated by dehydration, although it also can occur as a result of cold exposure.

Figure 94

Question 94 of 100
Figure 94 is an arthroscopic view of the intercondylar notch of a right knee from an
anterolateral portal. After injury to the structure as indicated by the asterisks, which
examination test most likely will demonstrate an abnormal finding?
1- Lachman test
2- Pivot-shift test
3- Posterior drawer test
4- Posterolateral (PL) drawer test

PREFERRED RESPONSE: 2- Pivot-shift test

77
AAOS 2016 Sports medicine

DISCUSSION
The structure shown is the PL bundle of the anterior cruciate ligament (ACL), which is
tight near terminal knee extension. Biomechanical analysis suggests the PL bundle provides a
greater degree of rotational stability than the anteromedial bundle. The pivot-shift test
evaluates for rotational instability of the ACL, while the Lachman test assesses anterior-
posterior stability. The posterior drawer and PL drawer test findings are positive after a
posterior cruciate ligament tear and PL corner injury, respectively.

RESPONSES FOR QUESTIONS 95 THROUGH 98


1- Axillary nerve injury
2- Musculocutaneous nerve injury
3- Radial nerve injury
4- Glenoid fracture
5- Loss of reduction
6- Deltoid detachment
7- Chondrolysis
8- Recurrent instability
9- Propionibacterium acnes (P. acnes) infection
10- Staphylococcus epidermidis
11- Staphylococcus aureus
Select the complication listed above that is most commonly associated with the shoulder
arthroscopy scenarios described below.

Question 95 of 100
Arthroscopic capsular release of the shoulder through the anteroinferior and
posteroinferior axillary pouch
1- Axillary nerve injury
2- Musculocutaneous nerve injury
3- Radial nerve injury
4- Glenoid fracture
5- Loss of reduction
6- Deltoid detachment
7- Chondrolysis

78
AAOS 2016 Sports medicine

8- Recurrent instability
9- Propionibacterium acnes (P. acnes) infection
10- Staphylococcus epidermidis
11- Staphylococcus aureus

PREFERRED RESPONSE: 1- Axillary nerve injury

Question 96 of 100
Use of an intra-articular glenohumeral bupivacaine pump for postsurgical pain control
1- Axillary nerve injury
2- Musculocutaneous nerve injury
3- Radial nerve injury
4- Glenoid fracture
5- Loss of reduction
6- Deltoid detachment
7- Chondrolysis
8- Recurrent instability
9- Propionibacterium acnes (P. acnes) infection
10- Staphylococcus epidermidis
11- Staphylococcus aureus

PREFERRED RESPONSE: 7- Chondrolysis

Question 97 of 100
Use of a 5 oclock portal in an arthroscopic Bankart repair
1- Axillary nerve injury
2- Musculocutaneous nerve injury
3- Radial nerve injury
4- Glenoid fracture
5- Loss of reduction

79
AAOS 2016 Sports medicine

6- Deltoid detachment
7- Chondrolysis
8- Recurrent instability
9- Propionibacterium acnes (P. acnes) infection
10- Staphylococcus epidermidis
11- Staphylococcus aureus

PREFERRED RESPONSE: 1- Axillary nerve injury

Question 98 of 100
A common infection organism that occurs after shoulder arthroscopy, it rarely is seen in
infections of the lower extremity
1- Axillary nerve injury
2- Musculocutaneous nerve injury
3- Radial nerve injury
4- Glenoid fracture
5- Loss of reduction
6- Deltoid detachment
7- Chondrolysis
8- Recurrent instability
9- Propionibacterium acnes (P. acnes) infection
10- Staphylococcus epidermidis
11- Staphylococcus aureus

PREFERRED RESPONSE: 9- Propionibacterium acnes (P. acnes) infection

DISCUSSION
Shoulder arthroscopy is becoming increasingly popular. Although it is a safe procedure, it
is not without potential complications. Neurovascular injury is possible after creation of
standard arthroscopic portals. Particularly, creation of anteroinferior portals puts the axillary
nerve at risk. Creation of an inferior 5 oclock (right shoulder) portal has been proven

80
AAOS 2016 Sports medicine

particularly dangerous and should be avoided. Similarly, considering the axillary nerves
proximity to the inferior capsule, arthroscopic release of the capsule in the area also puts the
nerve at risk for injury.
Bupivacaine has been shown to be chondrotoxic, and several reports have linked the use
of intra-articular bupivacaine pumps to postsurgical chondrolysis of the shoulder. Deep
infections are rare complications after arthroscopy, with incidence reported to range between
0 and 3.4% in the literature. The shoulder is particularly susceptible to P. acnes, a gram-
positive, microaerophilic, nonspore-forming bacillus that is predominant in the pilosebaceous
follicles that are most prevalent in the head, neck, and thorax. P. acnes, however, is not a
common organism isolated from infections of the knee and lower extremity.

Question 99 of 100
A 19-year-old collegiate middistance runner has a 4-year history of bilateral leg pain. Pain
begins within 10 minutes after starting to run and is described as a tightness and cramping in
the front of the legs. Symptoms resolve within 15 to 20 minutes of running cessation. A
presumptive diagnosis of exercise-induced compartment syndrome (EICS) is made, and the
patient elects to undergo compartmental pressure testing. What is the strongest indication for
elective fasciotomy of the anterior compartment?
1- Resting anterior compartment pressure of 13 mm Hg
2- Numbness/tingling of the plantar foot during the exercise portion of the test
3- 1-minute postexercise anterior compartment pressure of 42 mm Hg
4- 5-minute postexercise anterior compartment pressure of 19 mm Hg

PREFERRED RESPONSE: 3- 1-minute postexercise anterior compartment pressure of 42


mm Hg

DISCUSSION
This clinical scenario describes a patient with EICS, marked by a nonphysiologic rise in
muscle compartment pressure during exercise. Pressure testing is the best currently accepted
method of diagnosis. Most physicians use the following criteria for diagnosis: resting pressure
higher than 15 mm Hg, 1-minute postexercise pressure higher than 30 mm Hg, or 5-minute
postexercise pressure higher than 20 mm Hg. Only 1-minute postexercise anterior
compartment pressure of 42 mm Hg meets these criteria. Neurologic symptoms in the plantar
foot would imply involvement of the posterior compartments and would not support the
diagnosis of anterior compartment involvement.

81
AAOS 2016 Sports medicine

45
Figure 100

Question 100 of 100


Figure 100 is the MR image of a 19-year-old man who sustains recurrent anterior shoulder
dislocations. The lesion shown occupies approximately 10% of the articular surface. What is
the most appropriate treatment?
1- Open distal tibial allograft reconstruction
2- Open reduction and internal fixation (ORIF) with cannulated screws
3- Arthroscopic coracoid transfer
4- Arthroscopic repair incorporating the bony component

PREFERRED RESPONSE: 4- Arthroscopic repair incorporating the bony component

DISCUSSION
The MR image shows a bony Bankart lesion involving less than 20% of the glenoid joint
surface. One series reported high success rates after arthroscopic treatment when the defect
was incorporated into the repair. Anterior bony deficiencies occupying more than 25% to 30%
of the glenoid joint surface treated with soft-tissue repair only are associated with high

82
AAOS 2016 Sports medicine

recurrence rates. In these patients, an open or arthroscopic coracoid transfer or distal tibial
allograft reconstruction should be considered. ORIF has been reported for treatment of large
acute glenoid rim fractures, but is not recommended for recurrent anterior shoulder instability
in the setting of a 10% glenoid rim fracture.

83

S-ar putea să vă placă și